Đến nội dung

tuananh2000 nội dung

Có 216 mục bởi tuananh2000 (Tìm giới hạn từ 02-06-2020)



Sắp theo                Sắp xếp  

#503540 Ảnh thành viên

Đã gửi bởi tuananh2000 on 02-06-2014 - 15:31 trong Góc giao lưu

attachicon.gifa.PNG

 

Mình là thằng mặc áo lạnh, người còn lại là tuananh2000 (http://diendantoanho...51-tuananh2000/) :))

Mấy chú thấy em đẹp trai k?  :luoi:  :luoi:  :luoi:




#522809 Ảnh thành viên

Đã gửi bởi tuananh2000 on 04-09-2014 - 20:57 trong Góc giao lưu

 

attachicon.gif 1972427_1478427425706213_1253627581_n.jpg

bên trái là mình(NTP),bên phải là NRC

attachicon.gif 1900093_273865279442113_1585978746_n.jpg

và trên là A-Q

 

                                                                                      NTP

Những anh thanh niên cứng của năm  :icon10:  :icon10:  :icon10:  :icon10:  :namtay




#617705 Topic về phương trình và hệ phương trình

Đã gửi bởi tuananh2000 on 29-02-2016 - 20:29 trong Phương trình - hệ phương trình - bất phương trình

Bài 299 : $\sqrt{(x+2)(2x-1)}-3\sqrt{x+6}=4-\sqrt{(x+6)(2x-1)}+3\sqrt{x+2}$

Bài 300 : $(3x+1)\sqrt{2x^{2}-1}=5x^{2}+\frac{3x}{2}-3$




#614688 Topic về phương trình và hệ phương trình

Đã gửi bởi tuananh2000 on 13-02-2016 - 11:38 trong Phương trình - hệ phương trình - bất phương trình

Bài 208 (trích từ bạn minhminh98 ) , mình không nhớ ở topic này có chưa nhưng thấy khá khó :

 

 $\left\{\begin{matrix}x^2y+x^2+1=2x\sqrt{x^2y+2} & \\ y^3(x^6-1)+3y(x^2-2)+3y^2+4=0 & \end{matrix}\right.$

Mình làm thử bài này thì thấy gặp vấn đề là 

1) Ở dữ kiện đầu có cho $x^2y+x^2+1=2x\sqrt{x^2y+2}$ sau khi xét các TH sẽ được $y=\frac{x^{2}\pm 2x-1}{x^{2}}$

2) Ở dữ kiện sau ta có thể phân tích nhân tử  $y^3(x^6-1)+3y(x^2-2)+3y^2+4=0$ thành $(x^2y-y+1) (x^4y^2+x^2y^2-x^2 y+y^2-2 y+4) = 0$ nhưng vì $x^4y^2+x^2y^2-x^2 y+y^2-2 y+4=0$ có nghiệm khá xấu nên mình cũng chưa biết xử lí như thế nào @@!




#617983 Topic về phương trình và hệ phương trình

Đã gửi bởi tuananh2000 on 02-03-2016 - 04:29 trong Phương trình - hệ phương trình - bất phương trình

ĐKXĐ: .....

Ta có pt (1) $\Leftrightarrow (5x^2+\frac{3}{2}x-3)-(3x+1)\sqrt{2x^2-1}=0$

$\Leftrightarrow 2(2x^2-1)-(3x+1)\sqrt{2x^2-1}+x^2+\frac{3}{2}x-1=0$

Đặt $\sqrt{2x^2-1}=a (a\geq 0)$ ta có:

$2a^2-(3x+1)a+x^2+\frac{3}{2}x-1=0$

$\Leftrightarrow (a-x+\frac{1}{2})(2a-x-2)=0$

Đến đây dễ rồi 

Từ dòng đỏ trên sao bạn có ý tưởng phân tích thành dòng đỏ dưới được vậy, mình vẫn chưa hiểu rõ cách suy luận của bài toán lắm  :icon6:




#504568 $\boxed{\text{Chuyên Đề}}$ Bất đẳng thức - Cực trị

Đã gửi bởi tuananh2000 on 06-06-2014 - 20:59 trong Bất đẳng thức và cực trị

chỗ này bạn biến đổi như thế nào thế??? :)

cái lớn hơn hoặc bằng là bdt dạng cộng mẫu đó , còn ở dưới thì phải là (a+b+c)^2 mà  :luoi:  :luoi:  :luoi:  :luoi:  :luoi:




#556830 $\boxed{\text{Chuyên Đề}}$ Bất đẳng thức - Cực trị

Đã gửi bởi tuananh2000 on 28-04-2015 - 21:08 trong Bất đẳng thức và cực trị

Cho a,b,c >0 và a+b+c=3. CMR:

$\sum (a+b^2)\leq 13+abc$

Đề hình như nhầm nếu như vậy thì dễ quá  :huh:

$\sum (a+b^{2})=3+\sum a^{2}\leq 3+(\sum a)^{2}=12<13$




#503360 $\boxed{\text{Chuyên Đề}}$ Bất đẳng thức - Cực trị

Đã gửi bởi tuananh2000 on 01-06-2014 - 21:04 trong Bất đẳng thức và cực trị

Bài 174:

Cho $abc=1$ và $a^3>36$. Chứng minh rằng: $\frac{a^2}{3}+b^2+c^2> ab+bc+ca$

$\frac{a^{2}}{3}+b^{2}+c^{2}> ab+bc+ca\Leftrightarrow a^{2}-3(ab+ac+bc-b^{2}-c^{2})> 0$ . Thật vậy, ta có $a^{2}-3(ab+ac+bc-b^{2}-c^{2})=\frac{3a^{2}}{4}+\frac{a^{2}}{4}+3b^{2}+3c^{2}-3ab-3ac+6bc-9bc=3(\frac{a^{2}}{4}+b^{2}+c^{2}-ab-ac+2bc)+\frac{a^{2}}{4}-9bc=     3(\frac{a}{2}-b-c)^{2}+\frac{a^{3}-36abc}{4a}> 0(do abc=1 và a^{3}>36)$




#503545 Topic tổng hợp các bài toán về phương trình nghiệm nguyên.

Đã gửi bởi tuananh2000 on 02-06-2014 - 15:57 trong Số học

Bài 222 : Giải phương trình nghiệm nguyên  $54x^{3}+1=y^{3}$

http://diendantoanho...54x31y3pt-no-z/




#580176 Topic post ảnh người yêu, bạn gái,...

Đã gửi bởi tuananh2000 on 09-08-2015 - 22:14 trong Góc giao lưu

Sau bao cảm xúc dồn nén bấy lâu ,tối nay em up ngay hình "Gấu" của em

Xinh vậy , mình thích điều này  :D  :like




#567827 Topic tổng hợp một số bất đẳng thức trong kì thi MO các nước

Đã gửi bởi tuananh2000 on 24-06-2015 - 14:07 trong Bất đẳng thức - Cực trị

Bài 151(Romania TST): Cho a,b,c>0. CMR: nếu $a^2+b^2+c^2=3$ thì

$\sum \frac{1}{a^2+b^3+c^2}\leq 1$

 

Ta xét $\frac{1}{a^{2}+b^{3}+c^{2}}=\frac{1}{b^{3}-b^{2}+3}$

Ta cm$\frac{1}{b^{3}-b^{2}+3}\leq \frac{1}{3}+\frac{b^{2}-1}{18}$ hay $(b-1)^{2}\left [ \frac{b(6-b^{2})+3-b^{2})}{18(b^{3}+3-b^{2})} \right ]$

Bất đẳng thức trên hiển nhiên đúng vì $b>0$ và $b^{2}\leq 3$

TT rồi cộng theo vế được $ĐPCM$

Dấu $'='$ xảy ra khi $a=b=c=1$




#567772 Topic tổng hợp một số bất đẳng thức trong kì thi MO các nước

Đã gửi bởi tuananh2000 on 24-06-2015 - 09:10 trong Bất đẳng thức - Cực trị

 

 

Bài 147 (Làm mạnh bài USAMO 2003). Cho ba số $a,\,b,\,c$ dương. Chứng minh rằng

\[\sum \frac{(2a+b+c)^{2}}{2a^{2}+(b+c)^{2}}+\frac{3(a^2+b^2+c^2)}{(a+b+c)^2} \leqslant 9..\]
 

Chuẩn hóa $\sum a=3$

Ta có $\frac{(2a+b+c)^{2}}{2a^{2}+(b+c)^{2}}+\frac{3a^{2}}{(\sum a)^{2}}=\frac{(a+3)^{2}}{3a^{2}-6a+9}+\frac{3a^{2}}{9}$

Ta cm $\frac{(a+3)^{2}}{3a^{2}-6a+9}+\frac{3a^{2}}{9}\leq 3+2(a-1)$ hay $(a-1)^{2}(a-6)a\leq 0$ ( luôn đúng với $0\leq a\leq 3$)

Cộng các vế với nhau ta đc $ĐPCM$

Dấu $'='$ xảy ra khi $a=b=c>0$




#569420 Topic tổng hợp một số bất đẳng thức trong kì thi MO các nước

Đã gửi bởi tuananh2000 on 02-07-2015 - 10:59 trong Bất đẳng thức - Cực trị

Bài 165 (Mediterranean Mathematical Competition 2009).Chứng minh rằng với mọi  $a,b,c $ dương ta luôn có:

$$\sum \frac{ab}{a^2+ab+b^2}\leq \frac{a}{2a+b}+\frac{b}{2b+c}+\frac{c}{2c+a}$$

Đặt $(a;b;c)=(\frac{x}{y};\frac{y}{z};\frac{z}{x})$ , đưa bđt cần cm thành

$\sum \frac{x^{2}}{x^{3}+x^{2}+1}\leq\sum \frac{x}{2x+1}$

Hay $\sum(x-1)^{2}(x+1)x\geq 0$  (đúng)

Vậy bất đẳng thức đc cm

Dấu đẳng thức xảy ra khi $a=b=c=1$ 




#569441 Topic tổng hợp một số bất đẳng thức trong kì thi MO các nước

Đã gửi bởi tuananh2000 on 02-07-2015 - 14:23 trong Bất đẳng thức - Cực trị

Thế thì $abc=1$ à

Mình cũng chẳng biết @@




#567780 Topic tổng hợp một số bất đẳng thức trong kì thi MO các nước

Đã gửi bởi tuananh2000 on 24-06-2015 - 09:49 trong Bất đẳng thức - Cực trị

Bài 145(China TST): Cho a,b,c >0. CMR: nếu a+b+c=3

$\frac{1}{a^2+b+c}+\frac{1}{b^2+c+a}+\frac{1}{c^2+a+b}\leq 1$

Ta xét $\frac{1}{a^{2}+b+c}=\frac{1}{a^{2}-a+3}$

Ta cm$\frac{1}{a^{2}-a+3}\leq \frac{1}{3}+\frac{a-1}{9}$ hay $\frac{(a-1)^{2}(a-3)}{a^{2}-a+3}\leq 0$ ( luôn đúng với $0\leq a\leq 3$)

TT với các hạng tử còn lại rồi cộng theo vế có $ĐPCM$

Dấu $'='$ xảy ra khi $a=b=c=1$




#504244 Topic: Các bài toán về tính chia hết

Đã gửi bởi tuananh2000 on 05-06-2014 - 18:25 trong Số học

Bài 1.Chứng minh rằng với mọi n nguyên dương ta đều có 

$A=5^n(5^n+1)-6^n(3^n+2^n)\vdots 91$

Bài  2.Tìm tất cả các cặp số nguyên p,q thỏa mãn phương trình sau:

 $5^{2p}+1997=5^{2p^2}+q^2$

Bài 1:$A=5^n(5^n+1)-6^n(3^n+2^n)=25^{n}+5^{n}-18^{n}-12^{n}=(25^{n}-18^{n})-(12^{n}-5^{n})$$=B(7)-B(7)$chia hết $7$.Lại có $A=5^n(5^n+1)-6^n(3^n+2^n)=25^{n}+5^{n}-18^{n}-12^{n}=(25^{n}-12^{n})-(18^{n}-5^{n})$$=B(13)-B(13)$ chia hết $13$. Do $(13;7)=1$ nên có $A$ chia hết cho $13.7=91$




#537079 Topic ôn luyện cuộc thi máy tính bỏ túi casio

Đã gửi bởi tuananh2000 on 10-12-2014 - 21:06 trong Các dạng toán khác

 

xác định hệ số

 

Xác định a, b, c, d biết P(1) = -2011; P(2) = -2084; P(3) = -2385; P(-1) = -2045. 
Cho mình hỏi có cách nào tính nhanh bài này ko
 

 

Theo mình bạn tính $a$ theo $P(1)+P(-1)$ rồi thay vào $P(2)$ và $P(3)$ nhé , có được $P(2)$ ; $P(3)$ và $P(1)$ là các phương trình 3 ẩn vào Casio giải bình thường nhé   :namtay




#507952 TOPIC Chuẩn Bị Cho Thi HSG Toán 8

Đã gửi bởi tuananh2000 on 20-06-2014 - 07:51 trong Chuyên đề toán THCS

giải hệ ra ta được (a;b;c)=(0;0;1) và các giao hoán của nó => p=1

Bạn nêu rõ cách làm được không?  :closedeyes:  :closedeyes:




#508092 TOPIC Chuẩn Bị Cho Thi HSG Toán 8

Đã gửi bởi tuananh2000 on 20-06-2014 - 21:04 trong Chuyên đề toán THCS

thì a,b,c luôn $\leq$1 mà nếu 1 cái lớn hơn thì sẽ ko có pt 2

Bạn c/m điều đó thử nào ? :mellow:  :mellow:




#500840 TOPIC Chuẩn Bị Cho Thi HSG Toán 8

Đã gửi bởi tuananh2000 on 22-05-2014 - 21:49 trong Chuyên đề toán THCS

áp dụng cái này có cần cm ko bạn ? mik thấy thầy hay bắt mik cm cái này

$(a+b+c)(\frac{1}{a}+\frac{1}{b}+\frac{1}{c}) \geq 9$ Cái này quá quen r` :D




#508088 TOPIC Chuẩn Bị Cho Thi HSG Toán 8

Đã gửi bởi tuananh2000 on 20-06-2014 - 20:49 trong Chuyên đề toán THCS

mình nghĩ là thế này: trừ 2 vế ta được 

$a^{2012}(a^{2}-a)+b^{2012}(b^{2}-b)+c^{2012}(c^{2}-c)$=0

=>$\inline \left\{\begin{matrix} a^{2}-a=0\\ b^{2}-b=0\\ c^{2}-c=0 \end{matrix}\right.$

rồi => thôi

Lỡ $a^{2}-a< 0$ còn $b^{2}-b> 0$ thì sao bạn . Vẫn xảy ra trường hợp này mà




#508260 TOPIC Chuẩn Bị Cho Thi HSG Toán 8

Đã gửi bởi tuananh2000 on 21-06-2014 - 20:41 trong Chuyên đề toán THCS

theo diricle ta có 2 cặp cùng dấu giả sử 2 cặp đầu cùng dấu

=> a^2-a và b^2-b cùng dấu

giả sử a^2-a>0=> a>1 hoặc a<0 th a>1 loại vì ko tm pt 2

tương tự ta có b<0

từ c^2-c<0=> 0<c<1 

=> a^2013 +b^2013+c^2013<1

p/s mình lười latex quá các bạn cứ biến < thành$\leq$là có thể tìm nghiệm của hệ luôn

Sao lại vậy nhỉ ?




#524129 Topic Đề thi THCS

Đã gửi bởi tuananh2000 on 12-09-2014 - 21:54 trong Tài liệu - Đề thi

Bài 7 : Xét rằng $A+B+C=\pi$ $\rightarrow A+B=\pi -C \Leftrightarrow tg(A+B)=tg(\pi -C)\Leftrightarrow \frac{tgA+tgB}{1-tgA.tgB}=-tgC \rightarrow$ $tgA+tgB+tgC=tgA.tgB.tgC$ ($ĐPCM$)

P/s: Bài trên áp dụng $tg(\pi -\alpha )=-tg\alpha$  :luoi:  :luoi:




#523992 Topic Đề thi THCS

Đã gửi bởi tuananh2000 on 11-09-2014 - 23:26 trong Tài liệu - Đề thi

10689733_1484590751808563_24866820663089

Bài 5: 

a) Gọi $CP\cap (O)=\left \{ F \right \}; NB \cap (O)=\left \{ G \right \}$

Ta có:$\widehat{GCA}=\widehat{GBA}$ (2 góc này nội tiếp và cùng chắn cung $AM$)

$\widehat{ACP}=\widehat{ABN}$ ( cặp góc có canh t/ ứ vuông góc)

Do đó , $\widehat{GCA}=\widehat{ACF}\rightarrow$ Cung $AG$ =Cung $AF$ nên $AK$ vuông góc $GF$

Xét $\bigtriangleup BMH$ ($H$ là trực tâm) có $CN$ vừa là đường cao vừa là phân giác nên là tam giác cân đỉnh $C$ . Từ đó có $N$ là trung điểm $GH$ . Cmtt thì $D$ là trung điểm của $HF$

$\rightarrow DN$ là đường trung bình của $\bigtriangleup GHF$ $\rightarrow NP$ song song với $GF$

Ta có $AK$ vuông góc với $GF$ mà $GF$ lại song song với $NP$ nên $AK$ cũng vuông góc $NP$ $(đpcm)

P/s : câu b) thì mình vẫn chưa nghĩ ra  :icon6:  :icon6: , mấy bạn tự vẽ hình nhá  :namtay  :namtay




#525408 Topic Đề thi THCS

Đã gửi bởi tuananh2000 on 20-09-2014 - 22:15 trong Tài liệu - Đề thi

4/

PT $\bar{abc}=(a+b)^2.c\Leftrightarrow \frac{10\bar{ab}}{(a+b)^2-1}=c$

Do đó: $\begin{bmatrix} a+b=1\\ a+b=9\\ a+b=11 \end{bmatrix}$

*) TH1: $a+b=1$ $\rightarrow a=1;b=0$ (kg thỏa mãn)

*) TH2: $a+b=9$.$VT=\frac{\bar{ab}}{8}\in N\Rightarrow \frac{a+1}{8}\in N\rightarrow a=7;b=2;c=9$

*) TH3: $a+b=11$. 

$VT=\frac{\bar{ab}}{12}\in N\rightarrow\frac{9a-1}{12}$ (Loại)

Vậy 729 là số cần tìm.

Cái phần trên đâu thỏa đkxd đâu mà xét nhỉ  :icon6:  :icon6: